免費論壇 繁體 | 簡體
Sclub交友聊天~加入聊天室當版主
分享
返回列表 发帖

[函数] 代数系统的三道题

本帖最后由 hbghlyj 于 2019-10-6 10:52 编辑

(1)设f是$\left\langle {{\Bbb Q}, + } \right\rangle $与$\left\langle {{{\Bbb Q}^{\text{*}}}, \times } \right\rangle $间的同构映射,证明:$f:x\to a^x,a\in{\Bbb Q}^{\text{*}}$
$\left\langle {{\Bbb Q}, + } \right\rangle $的单位元0在f之下的象必是$\left\langle {{{\Bbb Q}^{\text{*}}}, \times } \right\rangle $的单位元1,除此之外呢?
(2)给定整数n≥3,求复数集的所有n元子集S,具有性质:$\forall a,b \in S$(可以相同),$ab\in S$
例如n次单位根的集合
(3)设a,b是非零复数,且$\frac ab$不是实数。定义$L_{a,b}=\{ra+sb|s\in \mbb Z\},R_{a,b}=\{z|z\in\mbb C^*∧L_{a,b}=L_{za,zb}\}$,试求当a,b变化时所有可能的集合$R_{a,b}$

本帖最后由 hbghlyj 于 2019-10-6 18:47 编辑

(2)法①$1\du$若$0\notin S$,可证$S=\{z\in\mbb C|z^n=1\}$
对任意z$\in$S,考虑{$z,z^2,z^3,\cdots$},必存在正整数r,s,r>s,使得$z^r=z^s$,由消去律$z^{r-s}=1$,即存在正整数u使得$z^u=1$,特别地$\left|z\right|=1$.如果$z\in$S,则对任意整数l,存在正整数t使$l+ut\gt0$就有$z^l=z^{l+ut}\in S$.记z={$z_1,z_2,\cdots,z_n=1$},其中$z_k=e^{i\theta _k},\theta _k\in(0,2\pi]$,设$l\lt\frac{\theta _k}{\theta _1}\lt l+1,l\in \mbb N_+$,则0$\lt\theta _k-l\theta _1\lt\theta _1$
注意到$e^{i(\theta _k-l\theta _1)}$矛盾dead
设$\frac{\theta _k}{\theta _1}=m_n,m_n\in\mbb N_+$,则$\theta_1=\frac{2\pi}{m}$,记$\omega=e^{\frac{2\pi i}{m}}=z_1$dead
另一方面S$\supset${$\omega ,\omega ^2,\cdots,\omega ^m$}
对比两边元素个数得n=m
$2\du$若0$\in$S,易知S\{0}也有此性质,由$1\du$知$S=\{0\}\cup\{z\in\mbb C|z^{n-1}=1\}$
法②1$\du$设S={$z_1,z_2,\cdots,z_n$},任取$z_k\in S$,注意到$z_1z_k,z_2z_k,\cdots,z_nz_k$,由消去律它们互不相等,它们就是S的一个排列,因此$z_1\cdot z_2\cdot \cdots\cdot z_n=z_1z_k\cdot z_2z_k\cdot \cdots\cdot z_nz_k$,由消去律$z_k^n=1$
2$\du$同法①
这个证明类似于费马小定理/欧拉定理,揭示了整数的缩系对于同余关系和复数的单位根集的同构关系,它们都是n阶循环群:设G是n阶群,则对任意的元素n$\in$G都有$a^n=1_G$

TOP

回复 2# hbghlyj
(3)$L_{a,b}$是a,b的整系数线性组合,它对加减法封闭,$\emptyset\ne L\subset \mbb C,0\ne z\in \mbb C$,令zL={zu|$u\in L$},$R_{a,b}$={$z|z\in\mbb C^*\wedge zL=L$}
,可证:若$z_1,z_2\in R_{a,b}$,则$z_1z_2\in R_{a,b}$
$z_1z_2L_{a,b}=z_1(z_2L_{a,b})=z_1L_{a,b}=L_{a,b}$
可证:若$z\in R_{a,b}$,则|z|=1
设m=$\min\{|B||B\in L_{a,b}\wedge B\ne0\}$
则|z|m=m,|z|=1
可证:$|R_{a,b}|\leq6$
设m=|A|,任取$z_1,z_2\in R_{a,b}$,则$z_1A,z_2A\in R_{a,b}$,则$|z_1-z_2|\geq m$,$\arg(\frac{z_1}{z_2})\geq\frac{\pi}{3}$
$R_{a,b}$是有限集,由(2)可证:$|R_{a,b}|=n$
因为$1,-1\in R_{a,b}$,所以n=2,4,6
$R_{a,b}$的所有可能为
{1,-1}{1,-1,i,-i}$\{1,\omega ,\omega ^2\cdots\omega^5\}$

TOP

返回列表 回复 发帖